¿Probar que la escisión que preserva la isometría es similar a Killing?

(Si cree que esto, por ejemplo, no está bien expresado, ya comprende la solicitud de ayuda).

Teorema : Dada una variedad METRO equipado con una métrica gramo y que posee al menos una isometría no trivial ϕ generado por un campo de exterminio k , para METRO = METRO W , dónde W es un subconjunto abierto en METRO ϕ : METRO METRO es una isometría si y sólo si pag W ¯ , T pag ( pag ) METRO es paralelo a k ( pag ) .

Corolario : Las simetrías de W determinar las isometrías de METRO : si tangentes a W ¯ están en k i en todas partes tangente a W ¯ , k i sigue siendo un campo de exterminio de METRO .


Si no es realmente falso, esto puede ser conocido / trivial, pero no puedo encontrar una prueba, y me veo obstaculizado por la falta de conocimiento y notación al construir uno (por lo que la declaración podría no ser muy buena tampoco; faltan condiciones en el múltiple, Por ejemplo).

Habiendo pensado en esto en el contexto específico de los campos de muerte temporales del espacio de Minkowski (métrica lorentziana), el caso general (riemmaniano o lorentziano) mencionado anteriormente parecía plausible, pero lo que realmente necesito es una prueba del caso especial.

bocetos _ El teorema es que si se escinde una región de una variedad, si el límite de esa región sigue las curvas integrales de un campo Killing de la variedad original, ese campo es un campo Killing de la variedad resultante.

Demostración (por contradicción en el caso de los campos de exterminio temporales del espacio de Minkowski... métrica lorentziana). Asumir que ϕ : METRO METRO es una isometría; elige un punto pag W ¯ ; desde W es temporal, siempre hay un campo de vector Killing k Paralelo a T pag ( pag ) METRO ; elige algun otro q W ¯ , entonces la tangente en q es paralelo a k o no lo es: si no lo es, W ¯ debe intersecar las curvas integrales de K y, por lo tanto, la escisión rompe la biyección (al eliminar los puntos de la imagen) y no puede haber isometría en absoluto: una contradicción. De este modo W ¯ debe regirse por las curvas integrales de K. (Probablemente necesite una extensión/reformulación del teorema para el caso general porque no hay garantía de que haya un vector Killing en cualquier lugar tangente al límite de escisión)

Las respuestas pedagógicas serán doblemente bienvenidas: ¡una cosa es tener una respuesta y otra entenderla!

(republicado con mejoras menores de math.se)

Respuestas (1)

Me parece que tu pregunta no tiene tanto que ver con los campos de exterminio. Es una pregunta más general. Considere un campo vectorial suave X sobre una variedad suave (Hausdorff) METRO y suponga que el grupo de un parámetro de difeomorfismos locales ϕ asociado a X es global (lo que equivale a decir que X está completo ). En otras palabras, si X METRO la ecuacion diferencial

γ ˙ X ( t ) = X ( γ X ( t ) )
con condición inicial
γ X ( 0 ) = X
admite una solución máxima (única) γ X = γ X ( t ) definido para todos t R .

Existen condiciones suficientes para asegurar que ϕ es global (por ejemplo, sucede siempre que METRO es compacto).

Por aquí, ϕ : R × METRO ( t , X ) ϕ t ( X ) := γ X ( t ) METRO es suave y bien definido. Además

(1) ϕ 0 = i d

y

(2) ϕ t ϕ τ = ϕ t + τ para cada t , τ R .

El caso que está considerando también requiere que METRO está equipado con una métrica no degenerada gramo y X es un completo gramo -Matar campo vectorial.

En este caso cada ϕ t : METRO METRO es una isometría.

Bueno, volviendo al caso general, la siguiente proposición es válida.

PROPUESTA . Dejar A METRO sea ​​un conjunto abierto cuya frontera A es una codimensión suave- 1 subvariedad incrustada de la variedad suave METRO y X un campo vectorial suave y completo en METRO . Entonces los siguientes dos hechos son equivalentes.

(a) ϕ t ( A ) = A y ϕ t ( METRO A ¯ ) = METRO A ¯ para cada t R .

(b) X es tangente a A .

prueba _

(1) Probamos que no (a) implica no (b).

si es falso eso ϕ t ( A ) = A y ϕ t ( METRO A ¯ ) = METRO A ¯ para todos t , entonces debe existir un punto X 0 A tal que ϕ t 0 ( X 0 ) A o un punto X 0 METRO A ¯ tal que ϕ t 0 ( X 0 ) METRO A ¯ para algunos t 0 R . Suponga que lo primero es válido (lo último puede tratarse de manera similar). Asumir t 0 > 0 el otro caso es análogo. Ahora hay dos posibilidades para ϕ t 0 ( X 0 ) A . Uno es ϕ t 0 ( X 0 ) A y en este caso definir s := t 0 . La otra posibilidad es ϕ t 0 ( X 0 ) METRO A ¯ . En este caso, defina

s := sorber { t [ 0 , + ) | ϕ τ ( X 0 ) A , τ < t } .
Este número existe y es finito (porque el conjunto no está vacío, ya que contiene 0 , y t 0 < + es un límite superior), estrictamente positivo no mayor que t 0 , y otra vez ϕ s ( X 0 ) A .

(De hecho, si ϕ s ( X 0 ) A hay un barrio abierto de ϕ s ( X 0 ) completamente incluido en A de modo que ϕ τ ( X 0 ) A también para algunos τ > s lo cual es imposible para la definición misma de sorber , si ϕ s ( X 0 ) METRO A ¯ , dado que este conjunto está abierto, habría un vecindario abierto de ϕ s ( X 0 ) completamente incluido en METRO A ¯ de modo que ϕ τ ( X 0 ) A en algunos ( s ϵ , s ] lo cual es de nuevo imposible para la definición misma de sorber ; la única identificación de caso restante ϕ s ( X 0 ) A .)

Probemos que tal s (en ambas posibilidades) no puede existir si (b) es válido. En efecto, X | A es un campo vectorial completo suave bien definido en la variedad suave A y por lo tanto el problema de Cauchy asociado sobre A con condición inicial γ ˙ ( s ) = ϕ s ( X 0 ) A en t = s admite una solución completa completamente contenida en A También por t < s , pero esta curva ahora vista como una línea integral de X en METRO está determinada de forma única y sabemos por hipótesis que comienza en X 0 A encontrar una contradicción.

(2) Probamos que no (b) implica no (a).

Supongamos que (b) es falso y encontramos que (a) también es falso. Supongamos ahora que hay X 0 A tal que X ( X 0 ) es transversal a A . Como A es una variedad suave embebida, X es suave y no se desvanece en X 0 , no es muy difícil demostrar que existe un parche de coordenadas X 1 , X 2 , . . . , X norte alrededor X 0 en METRO ( norte = d i metro ( METRO ) ) tal que X 0 ( 0 , 0 , , 0 ) , A es la parte del plano X 1 = 0 contenida en la imagen de la carta, y las curvas integrales de X son las curvas R t ( t , X 2 , , X norte ) (ver el ANEXO final ). Desde que el avión se separa A de METRO A ¯ , es evidente que hay puntos en A que se trasladan a METRO A ¯ por ϕ y viceversa. Por lo tanto ϕ t ( A ) = A y ϕ t ( METRO A ¯ ) = METRO A ¯ para cada t R Es falso.

QED

Evidentemente, si X es un campo Killing completo, el resultado se refiere al grupo asociado de isometrías de un parámetro.


ANEXO . Demuestro aquí que

lema _ Si S es un incrustado norte 1 subvariedad suave -dimensional de la norte -variedad lisa dimensional METRO , y X es un campo vectorial suave sobre METRO que no desaparece en X 0 S y no es tangente (es decir, es transversal) a S en X 0 , entonces hay un parche de coordenadas X 1 , X 2 , . . . , X norte alrededor X 0 en METRO tal que X 0 ( 0 , 0 , , 0 ) , S es la parte del plano X 1 = 0 contenida en la imagen de la carta, y las curvas integrales de X son (restricciones alrededor t = 0 de) las curvas R t ( t , X 2 , , X norte ) .

prueba _ Como S está incrustado, hay un parche de coordenadas ( tu , ψ ) en METRO alrededor X 0 S tal que ψ ( S tu ) = { ( y 1 , , y norte ) ψ ( tu ) | y 1 = 0 } y siempre podemos suponer ψ ( X 0 ) = ( 0 , , 0 ) . Ahora X = a Y a y a es tal que Y 1 ( 0 , , 0 ) 0 simplemente porque X es transversal a S en X 0 (las coordenadas y 2 , , y norte son coordenadas en S ). Las líneas integrales de X en coordenadas satisfacer d y a d t = Y a ( y 1 ( t ) , , y norte ( t ) ) . Somos libres de arreglar t = 0 exactamente en S para todas las curvas. Ahora introduce las coordenadas X 2 = y 2 , , X norte = y norte en S y escribir dichas curvas integrales como funciones suaves y k = y k ( t , X 2 , , X norte ) , dónde X 2 , , X norte denota el punto inicial en S (en t = 0 ) de la curva integral considerada. Dicho mapa es suave, como bien se sabe de los teoremas estándar sobre la dependencia suave de los datos iniciales de los problemas de Cauchy. Definir finalmente X 1 = t . Dado que la matriz jacobiana j = [ y a X b ] exactamente en X 0 satisface

det j ( X 0 ) = y 1 t | X 0 = Y 1 ( 0 , , 0 ) 0
El teorema de Dini demuestra que X 1 = t , X b = y b definir un sistema de coordenadas suave admisible en METRO alrededor X 0 . En coordenadas locales X 1 , , X norte , la porción de S entrar en el dominio de las coordenadas todavía está representado por X 1 = 0 (porque X 1 = t y todas las curvas integrales se intersecan S en t = 0 ) y, localmente, las curvas integrales de X son restricciones triviales alrededor t = 0 de las curvas R t ( t , X 2 , , X norte ) .

¡Un reconocimiento inicial rápido y muchas gracias! Tendrá que estudiar eso con cuidado, pero probablemente lo acepte más tarde.
Solo 1 pregunta inicial: Proposición (a): está eliminando el cierre de (abierto) A, ¿sí? ¿No deja eso a M sin límite? Tenía la intención de eliminar A, no su límite también. NB si está disponible en el chat en cualquier momento durante el fin de semana, hágamelo saber.
Elimino también el límite. La imagen es así simétrica en el intercambio de A y M. Lo siento, estoy demasiado ocupado y no puedo chatear.
Agregué algunos detalles más...
@Julian Moore ¿Qué pasa con los 50 puntos de la recompensa?
Aunque todavía tengo un par de preguntas, he aceptado la respuesta, con muchas gracias. Después de un poco de estudio, descubrí que la respuesta se basa en los fundamentos de la teoría de la mentira ... así que tengo una entrada, pero todavía soy matemáticamente ignorante, así que (si está inclinado a responder) P1: ¿Cómo se transfiere la prueba a M \ A en lugar de M\clos(A)? (Esto es clave para mí y no veo cómo ayuda la simetría de apertura entre fusiones y adquisiciones). Y los 3 intermedios: "no es difícil de probar", "es fácil", "trivial": ¿dónde podría encontrar ( en orden descendente de importancia!) ¿Más orientación sobre cómo completar los detalles? De nuevo, ¡mi más sincero agradecimiento!
¿Generosidad? Pensé que se otorgaba tan pronto como acepté. Lo comprobaré cuando llegue a casa.
Pregunté sobre esto en meta; Si bien es posible que su respuesta llegue justo al final del período de recompensa, considero que la culpa es mía debido a la ignorancia de los detalles del proceso de recompensa y, a modo de recompensa, ofreceré una recompensa adicional (cuando haya descubierto cómo ) de 50 ya que el resultado es muy importante para mi y valoro mucho tu respuesta; considerando que la culpa es mía, será completamente incondicional, pero cualquier aporte adicional que pueda ofrecer será muy apreciado. Disculpas por los inconvenientes. ¡También vi que @Alfred-Centauri ha agregado una recompensa adicional a modo de reconocimiento!
No te preocupes. Desafortunadamente, estoy bastante ocupado con varias cosas para agregar más detalles a mi respuesta, lo intentaré más tarde (tal vez el fin de semana) ...
OK @Julian Moore Encontré el tiempo para agregar algunas pruebas adicionales. Déjame saber si es mejor ahora...
Eres muy amable y comprensivo con el tema de las recompensas; ¡Sin embargo, persistiré con eso! Su respuesta es realmente ejemplar. ¡Ahora puedo estar en condiciones de completar mi trabajo!
Solo puedo agregar una recompensa cuando expire la recompensa actual y debe ser el doble, lo cual estoy más que feliz de dar.
Sin problemas, finalmente obtuve los 50 puntos de la recompensa (agregados por Alfred Centaury) ¡uno es suficiente!
No obstante... como prometí, y como una cuestión de principio y buena fe, he agregado mi propia recompensa de 100 por su considerable esfuerzo (con solicitud pero no condicionada a la extensión a A cerrado... es suficiente decir que sigue para A cerrado por simetría ya que no se confía en el conjunto al que pertenece el límite??)
Indique su consulta de forma clara e intentaré responder.
muchas gracias Quisiera saber cómo se puede emplear la demostración de la Proposición para obtener: la equivalencia de: (a') ϕ t ( A ) = A y ϕ t ( METRO A ) = METRO A para cada t R , y (b), para A METRO un conjunto abierto. Creo que veo cómo hacerlo, pero estoy lejos de tener confianza.
Es claro que (a) -> (a') porque, dado que A , METRO A ¯ , A son conjuntos disjuntos cuya unión s METRO , por diferencia (a) implica ϕ t ( A ) A y por lo tanto ϕ t ( A ) = A porque cada ϕ t es biyectiva. Supongamos ahora que se cumple (a'). Si X no es tangente en X 0 A , explotando el mismo argumento que usé en (2) puedes construir una línea integral que salga de A pasando por X ( X 0 ) . Entonces (a') implica en particular que X es tangente a A y así (b) se cumple, así (a')->(b) <->(a).
Resumiendo (a) -> (a') -> (b) -> (a) y así (a), (a') y (b) son equivalentes por pares.
Refiriéndose a (a)->(a') estaba un poco implícito. Para ser más explícito: tan pronto como sepas que ϕ t ( A ) = A , de la hipótesis (a) también se tiene ϕ t ( METRO A ) = ϕ t ( METRO A ¯ A ) = ϕ t ( METRO A ¯ ) ϕ t ( A ) = METRO A ¯ A = METRO A . Esto completa la demostración de que (a)->(a').
Eso fue lo que creo que pensé;) Muy bien hecho, y una vez más, ¡muchas gracias!
"bien conocido por los teoremas estándar sobre la dependencia suave de los datos iniciales de los problemas de Cauchy". Preferiría citar teoremas específicos con aplicación directa. O al menos para saber qué cuenta como 'bien conocido' en este nivel... (¿Cauchy-Kovalevski sería uno de ellos?) ¿Puedes ayudar con algunos detalles?
Bueno, no es más que el teorema que ya he usado para definir ϕ t ( X ) . La solución de una ecuación diferencial de primer orden definida por un campo vectorial suave en una variedad suave es una función suave conjunta del parámetro a lo largo de las soluciones y la condición inicial.
Hola Valter: una pregunta no técnica, si se me permite; obviamente, la afirmación original funcionó, pero tengo curiosidad por evaluar mi "intuición". ¿Diría que el boceto de prueba también estuvo bien motivado o pasó por alto factores cruciales? (Para su información, aplicación a una variedad M: colocar una métrica de firma de Lorentz en M compacto es equivalente (consulte, por ejemplo, Geroch J. Math. Phys '67 para obtener detalles) a colocar un campo vectorial continuo en M, por lo que para M una sección delimitada en forma de espacio de espacio-tiempo de Minkowski, la proposición es directamente aplicable, ¡como esperaba!)
Hola Julian, no era solo un comentario quería que se entendiera bien tu pregunta original. Sin embargo, sí, creo que su idea general era correcta, de hecho, lo que hice no fue más que expandir su idea a un contexto más general. (Le escribo desde mi teléfono, espero que se entienda).
Hola Valter, también al teléfono y disfrutando de una cerveza bien merecida! Su comentario anterior parece haber desaparecido, pero vale la pena decir que la "expansión" valió la pena. Nb, no mencioné el parámetro porque no conocía los difeomorfismos de 1 parámetro como tales;) Tiendo a aprender sobre esas cosas solo cuando las necesito, como en este caso;)
Mientras aplicamos LaTeX y reorganizamos la prueba... "Sea A⊂M un conjunto abierto cuyo límite ∂A..." ¿Es necesario reformularlo? El límite de un conjunto abierto es el conjunto vacío, ¿no es así? ¿Debería referirse al cierre? ¡Gracias!
El límite (o frontera) de la bola abierta { X R norte | | | X | | < 1 } es { X R norte | | | X | | = 1 } y no el conjunto vacío. El límite de un conjunto puede no tener intersección con el propio conjunto.
Leí la declaración incorrecta que cité en algún lugar aparentemente autorizado, pero confío más en usted que en "Internet";) Gracias.
Probablemente haya encontrado la definición de "variedad con límite", donde, por definición, el límite es parte del conjunto y existe una interacción precisa (no solo topológica) entre el volumen y el límite. En su lugar, utilicé la definición topológica general de límite, que es completamente suficiente para abordar su problema.